Đến nội dung

Visitor nội dung

Có 63 mục bởi Visitor (Tìm giới hạn từ 10-06-2020)



Sắp theo                Sắp xếp  

#635751 Marathon số học Olympic

Đã gửi bởi Visitor on 26-05-2016 - 20:40 trong Số học

@Visitor: Có một chỗ chưa chuẩn là điều kiện phải là $\left\lfloor (i + 1)\alpha \right\rfloor \ge n + 1$, lúc này thì đoạn cuối không có vô lí

Dấu "=" xảy ra khi $(i+1)\alpha =n+1$, chỉ cần chọn $\alpha$ vô tỉ thì ko xảy ra được 




#635739 Marathon số học Olympic

Đã gửi bởi Visitor on 26-05-2016 - 19:40 trong Số học

Bài 9.

Hình như mình hiểu nhầm đề bài hay sao nhỉ @_@ mà lại chứng minh được là chia được

Giả sử ko chia được như đề bài tức là sẽ có số $n$ ko thuộc vào $S(\alpha ),S(\beta ),S(\gamma )$

Khi đó tồn tại $i$ sao cho $i\alpha <n$ và $(i+1)\alpha >n+1$ $\Rightarrow \frac{i}{n}< \frac{1}{\alpha }< \frac{i+1}{n+1}$

tương tự tồn tại $j$ : $\frac{j}{n}< \frac{1}{\alpha }< \frac{j+1}{n+1}$

Cộng vế ta được $\Rightarrow \frac{i+j}{n}< \frac{1}{\alpha }+\frac{1}{\beta  }< \frac{i+j+2}{n+1}$

chọn $ \frac{1}{\alpha }+\frac{1}{\beta  } = 1$ thì $ \frac{i+j}{n}<1< \frac{i+j+2}{n+1}\Rightarrow i+j<n<i+j+1$

Vô lí. Vậy mọi số tự nhiên đều thuộc $S(\alpha ),S(\beta )$, $\gamma$ tùy ý 

Chắc mình nhầm ở đâu @"@




#635671 Marathon số học Olympic

Đã gửi bởi Visitor on 26-05-2016 - 12:58 trong Số học

Lời giải bài 5:

Ta sử dụng bổ đề quen thuộc : $v_{p}((qp^k+r)!)=v_{p}(qp^k)+v_{p}(r)$ với $r<p^k$, $p$ là số nguyên tố, $q,r$ là số tự nhiên.

C/m: $v_{p}((qp^k+r)!)=\frac{qp^k+r-S_{p}(qp^k+r)}{p-1}$

Do $r<p^k$ nên $r$ ko thể có biểu diễn quá $k$ chữ số trong hệ cơ số $p$ nên $S_{p}(qp^k+r)= S_{p}(qp^k)+S_{p}(r)$

suy ra: $v_{p}((qp^k+r)!)=\frac{qp^k+r-S_{p}(qp^k)-S_{p}(r)}{p-1}= \frac{qp^k-S_{p}(qp^k)}{p-1}+\frac{r-S_{p}(r)}{p-1}=v_{p}((qp^k)!)-v_{p}(r!)$ (đpcm)

Trở lại bài toán: 

Xét khai triển một số $n$ bất kì $n=p_{1}^{a_1}p_{2}^{a_2}...p_{k}^{a_k}.Â$

Theo $mod d$ thì ta thấy sẽ có hữu hạn bộ số $(a_1,a_2,...a_k)$ , cụ thể là $d^k$ bộ. Vì thế khi $n$ tăng đến vô cùng thì sẽ có vô số bộ bị lặp lại theo $modd$.

Xét các số dạng $a_s=(p_1p_2...p_k)^s$, theo nhận xét trên thì sẽ tồn tại các số $n_1<n_2<...<n_d$ sao cho : 

$v_{p_t}((a_{n_i})!)\equiv v_{p_t}((a_{n_j})!)(mod.d)$ với mọi $t$ từ $1$ đến $k$ và mọi $1\leq i,j\leq d$.

Vì các $n_i$ có thể lớn tùy ý nên ta chọn sao cho các điều kiện của bổ đề được thỏa mãn:

$a_{n_1}<min(p_{t}^{n_2})$

$a_{n_1}+a_{n_2}<min(p_{t}^{n_3})$

$a_{n_1}+a_{n_2}+a_{n_3}<min(p_{t}^{n_4})$

$........$

Áp dụng bổ đề ta có với mọi $p_t$ thì: 

$v_{p_t}((a_{n_2}+a_{n_1})!)=v_{p_t}((a_{n_2})!)+v_{p_t}((a_{n_1})!) \equiv 2v_{p_t}((a_{n_1})!)   (mod .p_t)$

$v_{p_t}((a_{n_3}+a_{n_2}+a_{n_1})!)=v_{p_t}((a_{n_3})!)+v_{p_t}((a_{n_2}+a_{n_1})!)\equiv 3v_{p_t}((a_{n_1})!) $

$........$

$v_{p_t}((a_{n_d}+...+a_{n_2}+a_{n_1})!)\equiv dv_{p_t}((a_{n_1})!) \equiv0$

CHọn $n=a_{n_d}+...+a_{n_2}+a_{n_1}$ thì $n$ thỏa đề. 
Vì $n_i$ lớn tùy ý nên cũng sẽ có vô số $n$ thỏa mãn đề bài.đpcm.

#

CHo mình nợ bài 6 vài phút.

Chả biết chọn bài nào, chọn đại 1 bài vui vui vậy :)

 

 

Bài toán 6. ( Toán cổ) Tìm tất cả số tự nhiên $n$ để có thể phân hoạch tập $S=\{1,2,3...,4n\}$ thành các tập con $4$ phần tử sao cho trong mỗi tập tồn tại một phần tử bằng trung bình cộng của ba phần tử còn lại.




#637064 Marathon số học Olympic

Đã gửi bởi Visitor on 31-05-2016 - 10:38 trong Số học

Lời giải bài $30$ 
Chọn $n=16.a^{8}$ 
$n^2+n+1=(4a^4-2a^2+1)(4a^4+2a^2+1)(4a^4-4a^3+2a^2-2a+1)(4a^4+4a^3+2a^2+2a+1)$

Ta chỉ cần quan tâm đến việc chọn $4a^4+4a^3+2a^2+2a+1,4a^4+2a^2+1$ như thế nào là đúng đắn nhất 
Chọn $a \equiv 1 \pmod{7},a \equiv 1 \pmod{13}$ hay $a \equiv 1 \pmod{91}$ 
Khi đó $\sqrt{n}=4a^4>\frac{4a^4+4a^3+2a^2+2a+1}{13},\frac{4a^4+2a^2+1}{7}$ 
Như vậy các ước nguyên tố sau luôn sẽ bé hơn mấy số này 
P/s : Khá khó khăn trong việc chọn modulo 

Nếu đã phân tích được thế kia thì chọn modulo có khó khăn gì đâu em. Cho anh hỏi em biến đổi biểu thức kia ntn thế?   quá khủng




#636996 Marathon số học Olympic

Đã gửi bởi Visitor on 31-05-2016 - 01:24 trong Số học

Lời giải bài 29.

Giả sử tồn tại $P(x),Q(x)$

Do $lim \pi(x)=+\propto$ nên $degP>degQ$

Mà $lim \frac{\pi(x)}{x}=0\Rightarrow lim\frac{P(x)}{xQ(x)}=0\Rightarrow degP<degQ+1$
Mâu thuẫn. Vậy ta có đpcm.

Bài 30. Chứng minh rằng tồn tại vô số số tự nhiên $n$ sao cho tất cả ước nguyên tố của $n^2+n+1$ đều ko lớn hơn $\sqrt{n}$




#637500 Marathon số học Olympic

Đã gửi bởi Visitor on 01-06-2016 - 22:03 trong Số học

 

 

Bài 37. [AoPS] Với mỗi số nguyên $r$, chứng minh rằng tồn tại số tự nhiên $n_r$ sao cho với mọi số nguyên $n>n_r$ tồn tại ít nhất một số nguyên dương $k$ thoả mãn $1 \le k \le n-1$ và $p^r$ là ước của $\binom{n}{k}$ với $p$ là số nguyên tố nào đó.

Lời giải của mình.

Lấy $p,q$ là hai số nguyên tố bất kì.

Với mỗi $r$ chọn $n_r=max(p^r,q^r)+100$

Xét số nguyên $n$ bất kì lớn hơn $n_r$. Ta thấy luôn luôn có $p^a-1\neq q^b-1$ do $p\neq q$ nên $n$ ko thể có cả hai dạng $p^a-1$ hoặc là $q^a-1$

Giả sử $n\neq p^a-1$, khi đó biểu diển $p$-phân của $n$ sẽ có chữ số tận cùng khác $p-1$.

Ta sẽ sử dụng định lí $Kummer$

Giả sử $n=(b_tb_{t-1}...b_1b_0)_p$ với $b_0\neq p-1$ và do cách chọn $n_r$ nên $t>>r$.

CHọn $k=[b_rb_{r-1}...b_1(b_0+1)]_p$ 

Dễ thấy rằng khi trừ $n-k$ trong hệ $p$-phân sẽ được kết quả là một số có $r+1$ chữ số tận cùng bằng $p-1$ ( $p-1$ là chữ số lớn nhất trong hệ cơ số này)

Khi đó thì phép cộng $n-k$ với $k$ sẽ nhớ ít nhất là $r$ lần, theo định lí $Kummer$ ta có $v_p\binom{n}{k}\geq r$.

Ta có đpcm.

 

Bài 40.   Tìm tất cả các số tự nhiên $m$ sao cho với mọi sô thuộc tập $\{0,1,2,..,m-1\}$ đều đồng  dư với một số dạng $x^2+y^2$ mod $m$.




#607597 Đề thi và lời giải VMO 2016

Đã gửi bởi Visitor on 06-01-2016 - 19:51 trong Thi HSG Quốc gia và Quốc tế

Co ban nào có ý tưởng cho cậu 2b ko?

Dễ thấy $lim(b_n-b_{n-1})=0$ mà $b_n$ lại tiến ra +vc nên ta có $[b_n]-[b_{n-1}]=0$ hoặc  $[b_n]-[b_{n-1}]=1$ với mọi $n$

Giả sử cái cần cm sai thì tồn tại $N$ để ${b_i} >= 1/2016$ với mọi $i>N$ . Đặt ${b_i} = {c_i} +1/2016$ thì $0< c_i < 2015/2016$

Do $lim(b_n-b_{n-1})=0$ nên từ chỉ số $m$ nào đó trở đi $b_n-b_{n-1}<1/2016$.

Chọn $k>max(N,m)+1$ và  $[b_k]-[b_{k-1}]=1$ thì có $b_k-b_{k-1}<1/2016$ suy ra $1+c_k - c_{k-1}<1/2016$ hay là $2015/2016+ c_k < c_{k-1} $

Vô lí. Vậy có $đpcm$

 

Cái $Fx$ của diễn đàn này bị lỗi rồi.




#635905 Marathon Tổ hợp và rời rạc VMF

Đã gửi bởi Visitor on 27-05-2016 - 12:04 trong Tổ hợp và rời rạc

Lời giải bài 8.


 

a/ thay các lá bài bằng các số từ $1$ đến $13$ . Anne sẽ gọi lần lượt từ số $1$ tới số $13$, lặp lại một số vòng gọi như vậy. Giả sử ô trống nằm giữa $2$ số $i<j$ thì khi gọi đến $i$,số $i$ sẽ chuyển vào chỗ trống. Khi đó sẽ có 2 số cạnh ô trống là $i$ và một số $k$ nữa. Nếu $k>i$ thì khi gọi đến $k$ $k$ k sẽ di chuyển vào ô trống, và $i$ bh ko cạnh ô trống nữa, còn nếu $k<i$ thì $k$ sẽ dịch chuyển vào ô trống trong vòng gọi thứ 2. Tóm lại $i$ sẽ ko ở cạnh ô trống nữa và sẽ có số khác nhảy vào ô trống. Như vậy sau 1 số bước thì các quân bài ko còn ở vị trí ban đầu nữa.

b/Để quân Át, tức số $1$ ko năm cạnh ô trống thì Anne chỉ việc gọi như phần a/ nhưng sẽ bắt đầu gọi từ $2$ đến $13$. Số $1$ sẽ đứng im còn ô trống sẽ thay đổi nên sẽ có lúc nào đó số $1$ ko nằm cạnh ô trống.




#631745 ĐỀ THI OLYMPIC CHUYÊN KHOA HỌC TỰ NHIÊN 2016

Đã gửi bởi Visitor on 07-05-2016 - 14:23 trong Thi HSG cấp Tỉnh, Thành phố. Olympic 30-4. Đề thi và kiểm tra đội tuyển các cấp.

 

ĐỀ THI OLYMPIC CHUYÊN KHOA HỌC TỰ NHIÊN 2016

Ngày 1 (07/05/2016)
 

Câu 4. Cho dãy số $(a_{n})_{n\in\mathbb{Z}^{+}}$ xác định như sau $$\begin{cases}a_{1} = 0 \\ a_{2} = 1 \\ a_{2n} = 2a_{n} + 1 \\ a_{2n + 1} = 2a_{n}\end{cases}$$ với mọi $n \in \mathbb{Z}^{+}$. Chứng minh rằng tồn tại vô số số nguyên dương $k$ sao cho $a_{k} = 2016$ và tìm số nguyên dương $k$ nhỏ nhất thỏa mãn điều này.
 

Câu này vui. CHuyển sang hệ cơ số 2 thì ta thấy: $a_{2n}$ sẽ bằng $a_n$ thêm số $1$ ở cuối, còn $a_{2n+1}$ bằng $a_n$ thêm số $0$ ở cuối. Mà $2$ số hạng bắt đầu là $0,1$ nên cứ thêm $1$ và $0$ vào cuối như vậy ta sẽ có mọi số tự nhiên :v

Còn $2016=(11111100000)_2$, cứ bỏ lần lượt các chữ số cuối đi là tìm được :v

 À còn vô hạn thì suy ra trực tiếp từ chuyện có vô hạn số $0$




#611857 $F_{10^{k}}$ $(k{\geq}1)$ luôn tận cùng bằng 5

Đã gửi bởi Visitor on 30-01-2016 - 22:59 trong Số học

Bài 1 (QVLuom) Cho vô hạn quả cân dạng $3^{i}$ với i=0,1,2,... mỗi loại 1 quả. Chứng minh rằng tồn tại một cách đặt duy nhất các quả cân lên chiếc cân đĩa sao cho với một vật có khối lượng k nguyên dương bất kì để cân thăng bằng.

Bài 2 (QVLuom) Chứng minh rằng tồn tại ít nhất 1 số nguyên tố nằm giữa 2 số chính phương liên tiếp $n^{2}$ và $(n+1)^{2}$ với $n{\geq}1$.

Bài 3 (Thelongduong) Chứng minh rằng số $F_{10^{k}}$ $(k{\geq}1)$ luôn tận cùng bằng 5 với $F_{n}$  là số Fibonacci thứ n

Bài 1 : nếu $k$ mà chia $3$ dư $2$ thì đặt quả cân kiểu gì :))
Bài 2 : ta có với $n$ đủ lớn thì ta có $(n+1)^2 < 2n^2 $ . Vậy là :)) nếu bài toán của tác giả là đúng thì nó sẽ mạnh hơn cả bổ đề $Bertrand$ :)) Điều này là ko thể 




#612379 $F_{10^{k}}$ $(k{\geq}1)$ luôn tận cùng bằng 5

Đã gửi bởi Visitor on 01-02-2016 - 23:45 trong Số học

vậy bạn có cm được bài 2 không? Hoặc co chứng minh được bổ đề đó không? Nhớ là tự làm nhé! Hoặc nếu theo ý kiến của bạn thì bạn có thể chứng minh bài 2 sai không? nếu làm không được thì đừng có nói nhé. Khoa học cần bằng chứng chứ không cần lời nói.

Mình ko đủ khả năng để chứng minh bài toán của bạn đúng hay sai. NHƯNG mình xin kể $1$ câu chuyện như sau :)

Có một định đề mang tên nhà toán học $Bertrand$ là : Với mọi $n>1$ thì giữa $n$ và $2n$ luôn có 1 số nguyên tố. Định đề này sau đó đã được $Chebyshev$ chứng minh.Cm cho định lí này rất khó. Cách sơ cấp nhất mà theo mình nhớ là được đưa lên báo TH và TT năm kia , trong đó dùng tới gần cả chục bổ đề nhỏ.

Như thế là đủ hiểu cm khó như nào.

Quay trở lại bài toán của bạn. Giả sử rằng nó đúng. như ở trên mình có viết là với $n$ đủ lớn thì $(n+1)^2<2n^2$ , tức là với $k$ đủ lớn ( tầm 25 trở đi) thì trong đoạn $[k,2k]$ sẽ chứa đoạn $[n^2,(n+1)^2]$ với $n$ nào đó. Mà theo bài toán của bạn thì đoạn này sẽ chứa ít nhất $1$ số nguyên tố. Do đó định lí $Bertrand$ đã được chứng minh.
Vậy bạn có thể đưa ra chứng minh cho bài toán $2$ để chúng ta có $1$ cách chứng minh rất mới cho định lí $Bertrand$  được ko :)

 

p/s : bạn ạ. diễn đàn là nơi trao đổi thảo luận về các bài toán :) dù mình ko làm được nhưng mình đưa ra ý kiến để thảo luận là đúng với ý chí của diễn đàn rồi. cách nói của bạn như kiểu đây là nơi chỉ để hỏi bài hoặc là đưa bài ra để thách thức vậy :)




#612375 $F_{10^{k}}$ $(k{\geq}1)$ luôn tận cùng bằng 5

Đã gửi bởi Visitor on 01-02-2016 - 23:16 trong Số học

thử trường hợp với $k=5$ nhé! ta đặt vật và quả cân $3^{0}$ và quả cân $3^{1}$ lên 1 đĩa đĩa kia đặt quả cân $3^{2}$ là được rồi :)

Ra là mình hiểu nhầm đề bài :) Vậy thì bài này mình quy nạp xem sao :) Giả sử bài toán đúng tới $k-1$ ta sẽ chưng sminh nó đúng với $k$

Bài toán đúng với $t$ thì ta kí hiệu $t-True$ :) . Kí hiệu tập các quả cân làm cho $t-True$ là $A_t$

Nếu $k=3t$ thì theo gtqn $t-True$ , lấy $A_k=3A_t$ là xong , từ đây ta có nhận xét là nếu $k-True$ và $k$ chia hết cho $3$ thì tất cả các quả cân trong $A_k$ đều nặng hơn $1$

Nếu $k=3t+1$ , lấy $A_k$ bằng cách thêm vào $A_{3t}$ quả cân $1$.

Nếu $k=3t+2$ thì ta làm lùi từ $h=3(t+1)$ bằng cách thêm vào đĩa chứa $h$ quả cân $1$ . Do $1$ chưa xuất hiện trong $A_h$ ( suy ra từ nhận xét ở trên)  .

Vậy ta có đpcm




#618560 Tìm $(x, y) \in \mathbb{Z}^{2}$:...

Đã gửi bởi Visitor on 05-03-2016 - 20:04 trong Số học

Lời giải của bạn rất hay nhưng có chỗ không đúng. Lý do: Ký hiệu Legendre chỉ dùng cho số nguyên tố lẻ thôi
Tuy nhiên vẫn có cách khắc phục: $y^{2} + 2y + 4 \vdots 2 \iff y \vdots 2 \iff x \vdots 2$. Mặt khác để ý $\text{VT} \equiv 2 \pmod{4}$ và $y^{3} \equiv 0\pmod{4}$. Vô lí, do đó $y^{2} + 2y + 4$ lẻ, áp dụng bài toán của bạn là xong.
P.s: mình có 1 ý tưởng khác xấu xí tí. :-). À Visitor này, mình hỏi riêng bạn chuyện này được không nhỉ?

lâu ko dùng Legendre quên mất :-) 
P.s: ừ,bạn ib đi :-)




#618437 Tìm $(x, y) \in \mathbb{Z}^{2}$:...

Đã gửi bởi Visitor on 05-03-2016 - 00:39 trong Số học

Tìm $(x, y) \in \mathbb{Z}^{2}$:
$$9x^{2} + 6x + 10 = y^{3}$$

$pt\Leftrightarrow (3x+1)^2+1=(y-2)(y^2+2y+4)$

Gọi $p$ là ước nguyên tố bất kì của $y^2+2y+4$ thì $p|(3x+1)^2+1$

$\Rightarrow (\frac{-3}{p})= 1,(\frac{-1}{p})=1\Rightarrow \Rightarrow  (\frac{3}{p})= 1$

suy ra $p\equiv \pm 1(mod12)\Rightarrow y^2+2y+4\equiv \pm 1(mod12)$ 

vô lí vì $ y^2+2y+4\not\equiv \pm 1(mod12)$

PT vô nghiệm 




#607654 Chứng minh rằng nếu $a,b \in \mathbb{X}$ thì...

Đã gửi bởi Visitor on 06-01-2016 - 22:15 trong Số học

:lol:Đã nhận ra lỗi.
Mình làm như sau: Gọi $a$ là số nguyên dương nhỏ nhất và $b$ là số nguyên âm lớn nhất của $\mathbb{X}$
Trường hợp 1: $a + b > 0 \implies a + b \ge 1 \implies a \ge 1 - b \ge 2$. Mặt khác $a + b \in \mathbb{X}$ và $a + b \in \mathbb{N} < a$ mâu thuẫn với sự nhỏ nhất của $a$.
Trường hợp 2: $a + b < 0$ (tương tự trên)
Trường hợp 3:(Cảm ơn bạn Zaraki đã nhắc nhở mình) $a + b = 0 \implies a = -b$. Khi đó mọi phần tử có dạng $Ma (\forall M \in \mathbb{Z})$ đều sẽ thuộc $\mathbb{X}$. Để ý $0$ cũng là phần tử của $\mathbb{X}$
- Nếu $a = 1$, thì $\mathbb{X} = \mathbb{N}$. Do đó điều cần chứng minh đúng.
- Nếu $a > 1$, gọi $K$ là một phần tử không chia hết cho $a$ của $\mathbb{X}$ (giả sử $K$ dương và có $K > a$ và $1 \le gcd(K; a) < a$). Để ý là khi đó sẽ suy ra tổ hợp tuyến tính $mK + na$ cũng thuộc $\mathbb{X}$. Do đó theo bổ đề Bezout thì $mK + na = gcd(K; a) < a \in \mathbb{X}$, mâu thuẫn với tính nhỏ nhất. Vậy $K$ phải có dạng $Ma$.
Tóm lại $\mathbb{X} = {Ma; M \in \mathbb{Z}}$, do đó có điều cần chứng minh.

Đến đoạn $a>1$ cũng có thể lấy ngay $k$ chia cho $a$ : $k=ma + r$ suy ra $r$ thuộc $X$ do đó $r=a$. Bezout cao siêu quá :)

Tên này có phải Tạ Nguyên ko. Thấy giống quá :)




#618438 CMR $\text{gcd}(2^{n} - 1; 3^{n} + 2)...

Đã gửi bởi Visitor on 05-03-2016 - 00:44 trong Số học

Cho $n$ là số nguyên dương. CMR $\text{gcd}(2^{n} - 1; 3^{n} + 2) = 1$

Bài này quá khó, bối rối quá. Ego post lời giải được ko.




#611980 Tìm GTNN của: P=$\frac{a^{3}+b^{3}+c^...

Đã gửi bởi Visitor on 31-01-2016 - 18:12 trong Bất đẳng thức - Cực trị

Cho $a,b,c> 0$.Tìm GTNN của: P=$\frac{a^{3}+b^{3}+c^{3}}{abc}+\frac{(a+b+c)^{2}}{a^{2}+b^{2}+c^{2}}$.

@@ 




#617208 CMR tồn tại số $x$ sao cho $a_1+x,a_2+x,...,a_n+x$ là số...

Đã gửi bởi Visitor on 27-02-2016 - 17:39 trong Số học

1/Cho $n$ số thực $a_1,a_2,...,a_n$ bất kì.Chứng minh rằng tồn tại số $x$ sao cho $a_1+x,a_2+x,...,a_n+x$ là số vô tỉ

 

 

Bài này cấp 2 đúng rồi :))

Phản chứng giả sử ko tồn tại $x$ . Gọi $t$ là một số vô tỉ bất kì

Xét bảng sau:

$t+a_1$           $t+a_2$            $t+a_3$ ...            $t+a_n$

$2t+a_1$         $2t+a_2$          $2t+a_3$ ...          $2t+a_n$

...

$nt+a_1$         $nt+a_2$          $nt+a_3$ ...          $nt+a_n$

$(n+1)t+a_1$   $(n+1)t+a_2$   $(n+1)t+a_3$ ...   $(n+1)t+a_n$

Từ giả thiết phản chứng ta suy ra mỗi hàng sẽ có ít nhất một số hữu tỉ, nên cả bảng sẽ có ít nhất $n+1$ số hữu tỉ.

Lại có $n$ cột nên tồn tại ít nhất hai số cùng cột, giả sử $2$ số hữu tỉ là $pt+a_k$ và $qt+a_k$.

suy ra $(p-q)t$ là hữu tỉ. Vậy vô lí. $đpcm$ 




#607642 CMR: Tồn tại các số tự nhiên a,b,c

Đã gửi bởi Visitor on 06-01-2016 - 21:54 trong Số học

CMR: Tồn tại các số tự nhiên a,b,c thỏa $a^{2}+b^{2}+c^{2}=3abc$ và Min(a,b,c)>2004

Giả sử $a,b,c$ là nghiệm ta sẽ tìm $t$ để cho $a,b,c+t$ cũng là nghiệm tức là $a^2 + b^2 + (c+t)^2 = 3ab(c+t)$ 

Vì $a,b,c$ là nghiệm nên ta đã có  $a^2 + b^2 + c^2 = 3abc$ nên rút ra được $t = 3ab-2c $

Đến đây thấy rằng nên chọn $c$ là số nhỏ nhất ngay từ đầu để đảm bảo $c+t$ dương.

Cứ làm như vậy các bộ nghiệm sẽ tăng dần lên và có $đpcm$ thôi 




#617212 CMR tồn tại số $x$ sao cho $a_1+x,a_2+x,...,a_n+x$ là số...

Đã gửi bởi Visitor on 27-02-2016 - 17:56 trong Số học

 

 

2/Tìm tất cả các số $n\in \mathbb{Z^+}$ có tính chất sau: Tập hợp $S=\{n;n+1;n+2;....;n+5\}$ có thể chia thành hai tập hợp con $X,Y$ sao cho tích các phần tử của $X$ bằng tích các phần tử của $Y$

Thôi đã ghé qua THCS rồi thì làm nốt :3

nếu chia được thì sẽ có một tập chứa $n$ do đó tích chia hết cho $n$ , rồi xét modulo $n$ hai vế , chắc hơi dài :v




#617215 CMR tồn tại số $x$ sao cho $a_1+x,a_2+x,...,a_n+x$ là số...

Đã gửi bởi Visitor on 27-02-2016 - 18:10 trong Số học

 

 

2/Tìm tất cả các số $n\in \mathbb{Z^+}$ có tính chất sau: Tập hợp $S=\{n;n+1;n+2;....;n+5\}$ có thể chia thành hai tập hợp con $X,Y$ sao cho tích các phần tử của $X$ bằng tích các phần tử của $Y$

cách $2$ cho bài $2$ :))

ta thấy rằng trong $6$ số tự nhiên liên tiếp chỉ nhiều nhất một số chia hết cho $7$.

Nếu trong tập $S$ có $1$ số chia hết cho $7$ thì giả sử số đó thuộc $X$. Khi đó tích các phần tử của $X$ sẽ chia hết cho $7$ còn $Y$ thì ko.Loại

Nếu trong $S$ ko có số nào chia hết cho $7$, thì khi chia tập $S$ cho $7$ sẽ được các số dư là $1,2,3,4,5,6$

Giả sử chia được thành $n(n+1)=(n+2)(n+3)(n+4)(n+5)$ ( cái này vô lí thật nhưng chỉ để tượng trưng thôi)

suy ra $[n(n+1)]^2= n(n+1)(n+2)(n+3)(n+4)(n+5)\equiv  1.2.3.4.5.6=6!\equiv -1$ $(mod 7)$

$\Rightarrow X^2+1\vdots 7$

vô lí vì mọi ước nguyên tố của $X^2+1$ chỉ có dạng $4k+1$

Vậy ko có $n$




#619306 Chứng minh tồn tại 3 điểm là ba đỉnh của 1 tam giác có tung và hoành độ của t...

Đã gửi bởi Visitor on 09-03-2016 - 12:18 trong Tổ hợp và rời rạc

trong mặt phẳng tọa độ Oxy, cho 19 điểm có hành độ và tung độ là các số nguyên, trong 19 điểm đó không có ba điểm nào thẳng hàng. chứng minh trong 19 điểm trên có ít nhất 3 điểm là 3 đỉnh của một tam giác có hoành độ và tung độ của trong tâm tam giác đó là các số nguyên..

Xét các điểm $(i,j)$ với $i,j \in  \left \{ 0,1,2 \right \}$ .Có tất cả $3*3=9$ điểm. Mà có $19$ điểm nên sẽ có ít nhất $3$ điểm cùng số dư với $1$ điểm $(i,j)$ khi chia cho $3$. $đpcm$




#618439 Chứng minh rằng mọi số nguyên lớn hơn 11 là tổng của hai hợp số.

Đã gửi bởi Visitor on 05-03-2016 - 01:25 trong Số học

 

2. Chứng minh rằng nếu  $f(x)=a_{n}x^N+a_{n-1}x^{n-1}+...+a_{1}x+a_{0}$ 

    là đa thức với hệ số nguyên thì tồn tại $y$ sao cho $f(y)$ là hợp số.

P/s:   Help me!! 

Giả sử $a_n> 0$ khi đó $limf(x)=+vc$. Vì để bài ko nói $y$ nguyên hay ko nên ta thấy $f(x)$ liên tục nên luôn tồn tại $y$ để $f(y)$ là hợp số =))

chơi lầy =))




#617272 $\gcd (n,1)+ \gcd (n,2)+ \cdots + \gcd (n,n) = 3n-3....

Đã gửi bởi Visitor on 27-02-2016 - 21:28 trong Số học

Hình như không đúng. Mình chứng minh được số cần tìm là chẵn và square-free, bị đứng một chỗ. Và bạn thử lại xem. $n = 2p$ luôn thỏa mãn với $p$ là số nguyên tố lẻ :-) ($n = 10, n = 14$ vẫn thỏa mãn)

thank Ego, nãy mình tính sai tổng xích ma ,đã edit  :)




#619426 Chứng minh tồn tại 3 điểm là ba đỉnh của 1 tam giác có tung và hoành độ của t...

Đã gửi bởi Visitor on 09-03-2016 - 21:55 trong Tổ hợp và rời rạc

bạn ơi.. điều quan trọng là cả tung lẫn hoành đều chia hết cho 3... vì vậy mà bạn cần xét kỹ hơn..bạn nên xem lại

bạn ơi... mình xét $(i,j)$ là đã xét cả tung hoành độ rồi mà...